Download D - VCOMcc

Survey
yes no Was this document useful for you?
   Thank you for your participation!

* Your assessment is very important for improving the workof artificial intelligence, which forms the content of this project

Document related concepts

Electrocardiography wikipedia , lookup

Heart rate wikipedia , lookup

Biofluid dynamics wikipedia , lookup

Cushing reflex wikipedia , lookup

Homeostasis wikipedia , lookup

Common raven physiology wikipedia , lookup

Circulatory system wikipedia , lookup

Haemodynamic response wikipedia , lookup

Hemodynamics wikipedia , lookup

Intracranial pressure wikipedia , lookup

Blood pressure measurement wikipedia , lookup

Cardiac output wikipedia , lookup

Blood pressure wikipedia , lookup

Transcript
Cell and Physio Practice Exam 2 – Block 5 (2013)
Equations provided in old exams:
 
P
h
ΔP = QR
Series: R1 + R2 + R3 + R4 +…Rn = Rt
Parallel: 1/R1 + 1/R2 + 1/R3 + 1/R4 +….1/Rn = 1/Rt
Parallel: C1 + C2 + C3 + C4 +…Cn = Ct
Series: 1/C1 + 1/C2 + 1/C3 + 1/C4 +….1/Cn = 1/Ct
Compliance = DV/DP
CO 
VR 
Q O2consumed
O2arterial  O2mixed venous
Psf  PRA
RVR
Where Psf = mean systemic filling pressure (MSFP)
1. Stimulation of vagus nerve (CN X) to the sinoatrial (SA) node will cause a(n)
A. decrease in heart rate with no change in the phase 0 slope of SA nodal cells.
B. decrease in the slope of phase 0 of SA nodal cells a decrease in heart rate.
C. decrease in the slope of phase 0 of the SA nodal cells and an increase in heart rate.
D. increase in heart rate with no change in the phase 0 slope of SA nodal cells.
E. increase in the slope of phase 0 of SA nodal cells and a decrease in heart rate.
Please use the following tabulated data in Table 1 to answer questions 10 through 14. Please note that not all data may be needed and that data may be used
more than once to answer a given question.
Table 1
Hemodynamic Parameter
Value
Aortic Systolic Pressure
120 mmHg
Aortic Diastolic Pressure
82 mmHg
R to R interval from EKG
800 msec
Left ventricular end diastolic volume
145 ml
Left ventricular end systolic volume
75 ml
Pulmonary capillary wedge pressure
10 mmHg
Right atrial mean pressure
4 mmHg
Left atrial mean pressure
8 mmHg
Whole body oxygen consumption
Oxygen content femoral artery
Oxygen content pulmonary artery
250 ml/min
0.20 ml/ml blood
0.152 ml/ml blood
2. Calculate the approximate stroke volume of the left ventricle in ml from the provided ventricular volumetric data.
A.
B.
C.
D.
E.
19.33
38
56.3
70
75
3. Calculate the approximate ejection fraction of the left ventricle as a percent from the provided ventricular volumetric data.
A.
B.
C.
D.
E.
48
52
64
145
155
4. Calculate the patient’s heart rate
A.
B.
C.
D.
E.
60
75
80
120
175
5. Calculate the cardiac output in liters from the provided ventricular volumetric data.
A.
B.
C.
D.
E.
2.85
3.36
4.22
5.25
5.63
6. Calculate the cardiac output in liters using the Fick principle.
A.
B.
C.
D.
E.
1.90
3.52
5.21
5.25
7.60
7. Afterload of the left ventricle is most like:
A. Aortic pressure.
B. Left atrial mean pressure.
C. Left ventricular end diastolic pressure.
D. The double product.
E. The pressure developed with the opening of the mitral valve.
In the figure below please identify the following
B
A
C
D
a
b
E
a
b
D
a
b
8. Left ventricular peak systolic pressure B
9. Left ventricular end diastolic pressure A
10. Left ventricular end systolic volume E
11. Left ventricular end diastolic volume AD
12. Which of the following statements regarding the Frank-Starling relationship is most accurate.
A.
B.
C.
D.
E.
As ventricular end systolic pressure (VESP) increases, ventricular end diastolic volume (VEDV) increases causing an increase in stroke volume.
Decreased ventricular compliance increases the effects of VEDP on cardiac performance.
The Frank-Starling mechanism matches venous return to cardiac output.
The underlying molecular mechanism is related to rate of K+ flux during sytole.
Ventricular end systolic pressure (VESDP) is a good index of preload.
13. The figure, above, shows two pressure-volume loops for two differing situations. When compared with loop A, loop B demonstrates
A.
B.
C.
D.
E.
increased preload.
decreased preload.
increased contractility.
increased afterload.
increased preload.
14. Using the figure on question 22, a comparison of loop A with loop B shows that with respect to loop A, in loop B the
A. end diastolic pressure is greater.
B. diastolic pressure is less.
C. stroke volume is increased.
D. stroke volume is decreased.
E. stroke volumes are identical.
(ml)
Using the labeled figure above, please match the following segments with the letter.
Event
Segment
15. Rapid ejection period C
a) A - C
16. Isovolumetric relaxation D
b) C - D
17. Isovolumetric contraction B
c) D - E
d) F - A
e) E - F
18. During administration of an unknown agent, the systolic pressure rises, the ventricular end-diastolic volume is unchanged, and the ventricular end-systolic
volume decreases. The drug was:
A. a pure arteriolar vasoconstrictor
B. a negative chronotrophic agent
C. a positive inotropic agent
D. a vasoconstrictor with negative inotropic properties
E. a blood volume expander.
19. An elderly female patient with a childhood history of rheumatic fever is found to have a stenotic (decreased diameter) mitral valve. Diagnostic studies
reveal a reduced left ventricular stroke volume that is most likely caused by which one of the following?
A.
B.
C.
D.
Decreased left ventricular preload.
Decreased myocardial contractility.
Increased left ventricular afterload.
Increased left ventricular compliance.
20. The Frank-Starling mechanism states that in the normal heart:
A.
B.
C.
D.
As venous return increases, force of contraction increases.
As venous return increases, the stretch on the heart often exceeds optimal myofibril overlap.
Increased venous return results in increased contractility by sympathetic stimulation.
Increased venous return results in increased heart rate only.
21. What is the approximate ejection fraction (EF) as a percent for data from the pressure-volume loop below.
A.
B.
C.
D.
25%
33%
50%
67%
22. Vitrial Maaden has her cardiac output (CO) measured at an initial state (solid line) and following an intervention (dashed line). A graph of the results are
shown to the right. Comparison of the two curves indicates:
A.
B.
C.
D.
E.
Decreased blood volume
Decreased CO or venous return in the steady state after the intervention.
Decreased resistance to venous return.
Increased mean systemic (peripheral) blood pressure.
Increased myocardial contractility
23. Sammy Rudepickle has a measured left ventricular end diastolic volume of 150 ml and an end systolic volume of 50 ml. His heart rate is 60 beats/min and
is mean arterial pressure is 100 mmHg. What is Sammy’s “double product”?
A.
B.
C.
D.
3000
3300
6000
10000
24. An increase in resistance to venous return with no change in mean systemic filling pressure and right atrial pressure will result in:
A.
B.
C.
D.
E.
Decreased mean capillary filling pressure.
Decreased venous return.
Increased mean capillary filling pressure.
Increased venous return.
No change in venous return.
25. A patient with hypertension has a calculated left ventricular radius of 2 cm at the widest point. When repeated a year later his left ventricular radius is
calculated to be 4cm at the widest point. Neither change in ventricular wall thickness nor pressure is noted. During the year from initial evaluation to
follow up wall stress:
A. Decreased by one half.
B. Doubled.
C. Increased by a factor of four.
D. Increased by a factor of sixteen.
E. Remained the same.
26. Which of the following is normally associated with an increase in venous return to the heart?
A. Acute large arterial dilation.
B. Decreased mean systemic filling pressure.
C. Increased blood volume.
D. Increased venous compliance.
27. Which of the following will cause an increase in myocardial oxygen consumption (mVO 2)?
A. Decreased aortic pressure.
B. Decreased heart rate
C. Decreased myocardial contractility.
D. Decreased preload.
E. Increased afterload.
28. What is the compliance of a vessel if the vessel changes from a volume of 10 to 20 ml when pressure is increased from 5 to 15 mmHg?
A. 0.5 ml/mmHg
B. 0.5 mmHg/ml
C. 1ml/mmHg
D. 1mmHg/ml
29. What is the cardiac output (CO) if mean systemic filling pressure (Psf) is 10 mmHg, right atrial pressure is 2 mmHg, and resistance to venous return is 1.6
mmHg/L?.
A. 0L
B. 1.25L
C. 5L
D. 6.25L
E. 7L
30. Which of the following is an extrinsic factor that may increase SV?
A. decreased LVEDV
B. decreased preload
C. increased adrenergic stimulation
D. increased afterload
E. increased heart rate
31. Dr. Reilly is given two agents and a large supply of laboratory rats. He divides the rat population into two groups. Both groups demonstrate statistically
identical measures before any intervention. Group A receives an agent that increases by 33% the SV. In group A neither a change in the heart rate nor
peripheral (systemic) systolic blood pressure was seen. Group B receives an agent that causes a 33% increase in systolic blood pressure but no change in
SV and heart rate. Which of the following statements best defines the cardiac physiology seen in this experiment?
A. Agent A increased oxygen consumption more than agent B.
B. Agent A increased the external work of the LV more than agent B.
C. Agent B increased preload while Agent A did not.
D. Agent B increased the internal work of the LV more than agent A.
E. Cardiac efficiency is greater with agent B than agent A.
32. Which of the following statements about cerebral circulation is true?
A. Excessive pressure in the brain case is removed by cerebral lymphatics.
B. Increased intracranial pressure increases neurological function.
C. Increases in blood vascular volume in a region must be met with a reciprocal decrease in another region or intracranial pressure will decrease.
D. Redundant arterial circulation insures that blood supply/pressure to brain is maintained and equilibrated.
E. The skull encloses the brain vasculature in an expandable container.
33. Which of the following statements best describes cerebral vascular regulation?
A. Arterial blood pressure is autoregulated from ~10 to 40 mmHg.
B. Brainstem vasculature exhibits the most precise autoregulation.
C. Cerebral cortical vasculature has the most precise autoregulatory ability.
D. The sagittal sinus contains baroreceptors responsive to mean cerebral vascular pressure that autoregulate blood flow.
E. Venous blood pressure is autoregulated and remains at greater than 40 mmHg .
34. Which one of the following statements regarding myogenic and metabolic regulation is true?
A. Cerebral arteries and arterioles are not inherently responsive to increased pressure.
B. Cerebral arterioles and arteries show disproportionately myogenic responses with arteries having a greater proportionate response than
arterioles.
C. Decreased K+o occurs when many action potentials are locally generated.
D. NO comes from endothelium, neuronal and glial components.
E. pH increases cause dilation and pH decreases cause constriction.
35. Which of the following statements about autonomic neuronal motor regulation of the cerebral vasculature is true?
A. Decreases in the set point for neuronal autoregulation to lower arterial pressures decreases BBB integrity.
B. Long term disruption of sympathetics degrades autoregulatory ability.
C. Neuronal regulation is minor and the presence of innervation is not needed to maintain system integrity.
D. Parasympathetic innervation arises from branches of CN V.
E. Sympathetic innervation arises from inferior cervical ganglia.
36. Which statement best describes the effects of chronic hypertension on cerebral circulation?
A. Autoregulation becomes impaired at low cerebral arterial pressures.
B.
Increased NO production by perivascular glial cells increases the local effects of chronic cerebral hypertension.
C. The autoregulatory range is adjusted downward by VSM atrophy.
D. The autoregulatory range is adjusted downward by VSM hypertrophy.
E. The cerebral vascular resistance decreases to allow for normal capillary perfusion pressures.
37. Which statement about coronary circulation is true?
A. During diastole blood flow in the LV myocardium goes to zero.
B. Myogenic and metabolic regulation does not apply to coronary arterioles.
C. NO production within the vasculature decreases with increased shear rate.
D. Vasodilatation occurs through b2 adrenergic receptors.
E. Within the RV blood flow goes to zero during systole.
38. Please arrange the following components of short term neuronal regulation in the correct order.
1. Afferent neuronal pathway
2. A detector
3. Return of peripheral signal to the set point.
4. Efferent pathway with an error signal to the effector
5. ANS control center compares signal peripherally generated with a set point
A. 1,2,3,4,5
B. 2,1,5,4,3
C. 5,3,4,2,1
D. 2,3,4,5,1
E. 5,1,3,4,2
39. The primary control of systemic mean arterial pressure is_________.
A. left ventricular baroreceptors.
B. medullary chemoreceptors
C. peripheral baroreceptors.
D. peripheral chemoreceptors.
E. right atrial chemoreceptors.
40. A patient with a heart rate of 67 is given 300 ml of blood over a minute and heart rate increases to 110 BPM. What is seen here is:
A. A reflex tachycardia caused by an decrease in central venous pressure (CVP).
B. A reflex tachycardia causes by an increase in CVP.
C. An increase in diastolic filling due to tachycardia.
D. An increased heart rate due to increased impulse frequency of the vagus
E. A phenomenon that is maximal at higher heart rates
41. Which of the following statements best describes the action of the rennin-angiotensin- aldosterone system (RAAS)?
A. Aldosterone causes a decrease in Na+ and H2O reabsorbsion by the kidney.
B. Angiotensin converting enzyme (ACE) is a product of the cerebral vasculature.
C. Angiotensin I is converted to angiotensin II by the enzyme renin.
D. Angiotensinogen is a profound vasodilator that when converted to angiotensin II is inactive.
E. RAAS responds to decreased renal arterial pressure by increasing the release of renin an enzyme that converts angiotensinogen to
angiotensin I.
42. A patient completes a graded, dynamic exercise test on a treadmill that shows a modest rise (25%) in mean arterial blood pressure at lower exercise levels.
In contrast, during the highest level of exercise at the end of the test, an indirect method shows that cardiac output has risen 300% from rest. These
results indicate that during graded, dynamic exercise to exhaustion, systemic vascular resistance___________.
A. decreases slightly
B. falls dramatically
C. increases markedly
D. is constant
E. rises slightly
43. A patient with inflammatory bowel disease asks if exercise will alter blood flow to the gastrointestinal tract. You answer that vasoconstriction in the
splanchnic vascular beds during exercise:
A. allows splanchnic flows to parallel cerebral blood flow.
B. helps maintain arterial blood pressure.
C. occurs only after prolonged training.
D. rarely occurs
E. will be balanced by local dilation in these vascular beds.
44. When compared with a normal resting value, in a normal individual which of the following would cause the greatest decrease in stroke volume?
A. A heart rate of 200 beats/min.
B. An increase in end-diastolic pressure.
C. Reduction in afterload.
D. Stimulation of sympathetic nerves to the heart.
E. Stimulation of the vagus nerves.
45. A 14 year old male is evaluated for sports participation. Physical examination revealed a pan-systolic thrill on cardiac auscultation. Echocardiography
suggested a stenotic aortic valve i.e., aortic stenosis (AS). What is the name of the principle demonstrated here effecting coronary blood pressure during
rapid ejection?
A. Bainbridge’s
B. Bernoulli’s
C. Cushing’s
D. Ohm’s
E. Karotkoff’s
46. With exercise the heart normally ejects a greater volume per unit time. How would this affect coronary arterial blood flow in patients with aortic stenois
compared to normal individuals?
A. Blood flow will adjust to cardiac work and the flow will increase and then match myocardial oxygen demands.
B. Blood flow will be greater as aortic blood velocity will be greatly increased.
C. Blood flow will be less as aortic blood velocity will be greatly increased.
D. There will be no change in coronary blood flow as the increased work of the heart will balance the increased velocity of aortic blood.
47. During exercise, heart rate is linearly related to:
A. Age
B. Body weight
C. Oxygen consumption
D. Stroke volume
48. Which of the following is not increased during aerobic exercise?
A. Heart rate
B. Respiratory rate
C. Systolic blood pressure
D. Total peripheral resistance
49. Constriction of which of the following will have the greatest influence on capillary pressure:
A. Aorta
B. Post-capillary venule
C. Pre-capillary arteriole
D. Small arteries
E. Small veins
50. During a femoral arterial catheterization a velocity of 50 cm/sec was calculated and the cross sectional area of the femoral artery was determined to be 2
cm2. What is the flow through this vessel?
A. 0.04 ml/sec
B. 100 ml/sec
C. 200ml/sec
D. 25 sec
E. 50 ml
51. A patient develops exertional angina and is referred by his family physician for a cardiovascular work up. A stress treadmill reveals significant ST
depression consistent with posterior septal ischemia. A coronary catheterization shows a 50% stenosis of the posterior descending coronary artery.
Blood flow from immediately before to immediately after the stenotic lesion will be_______________.
A. unchanged
B. one half
C. one quartet
D. one eighth
E. one sixteenth
52. If a patient develops anemia causing a 50% decrease in blood viscosity and no other conditions change then blood flow through the abdominal aorta will
be_____________.
A. doubled
B. halved
C. unchanged
D. quadrupled
E. tripled
53. A patient with hypoproteinemia (low plasma protein) will normally have:
A. Low lymphatic flow.
B. High lymphatic flow.
C. Normal lymphatic flow.
D. Higher than normal plasma osmotic pressure.
E. Signs of tissue dehydration.
54. As blood pressure in a capillary rises capillary closing pressure will:
A. Increase.
B. Decrease.
C. Show no change.
D. Match arterial pressure.
E. Match venous pressure.
55. The apparent viscosity of blood will:
A. Decrease as the number of lamina within a blood vessel increase.
B. Decrease as vessel diameters decrease from aorta to capillary.
C. Increase as vessel diameters decrease from aorta to capillary.
D. Increases with decreasing hematocrit.
E. Is unchanged from capillary to aorta.
Mr. Igor Lipoma is a 67 year old white male, 1.6 m in height and 86 kg in mass. While in hospital Mr. Lipoma complains of dyspnea on exertion. Mr. Lipoma
had a quadruple coronary arterial bypass a year ago and has an enlarged cardiac silhouette on x-ray. EKG shows a regular rate of 82 beats per minute and
evidence of a pervious myocardial infarction. Mr. Lipoma has normal laboratory values for blood and urine. His blood pressure is 105/90 while supine.
Measured oxygen consumption is 336 ml/min. His blood gasses are 196 ml/L oxygen measured at the brachial artery and 99 ml/L oxygen when measured
from a pulmonary arterial sample obtained simultaneously.
1)
Mr. Lipoma has a cardiac output of approximately ____ L/min.
a) 0.71
b)
1.41
c)
d)
2)
a)
b)
c)
2.39
2.87
Mr. Lipoma has a stroke volume of approximately ____ ml.
9
17
35
e)
3.46*
d)
e)
42*
58
Later that day Mr. Lipoma becomes bradycardic with a heart rate of 52 beats per minute. His cardiac output as measured by
thermodilution is 2.1 L/min. His blood pressure is 103/92 while supine.
3)
4)
1.
2.
a)
b)
c)
Mr. Lipoma has a stroke volume of approximately ____ ml.
19
25
40*
96
109
Comparison of the stroke volumes obtained while Mr. Lipoma had a heart rate of 82 with that when his rate was 52
beats per minute suggests that Mr. Lipoma has
a) a very compliant left ventricle.
E. Normal
b) atrial flutter.
c) a noncompliant left ventricle.*
d) increased cardiac reserve.
4. Blood pressure measured indirectly by using a
e) paradoxical nocturnal dyspnea
sphygmomanometer:
An 88-year-old white female is brought to the
emergency department after several episodes of
syncope (fainting) upon assumption of an upright
posture. During upright posture the greatest
pressure gradient is between the:
A. Aortic arch and right atria.
B. Femoral artery and femoral vein.
C. Aortic arch and femoral vein.
D. Femoral artery and left atria.
If the baroreceptor reflexes are fully functional
when a person suddenly changes from recumbent to
standing posture:
A.
B.
C.
D.
E.
3.
d)
e)
C.
D.
E.
5.
Arterial pressure in the foot will be
maintained at 120/80 mmHg.
Blood vessels in the arm will become
vasodilated.
Bradycardia will rapidly occur.
Cerebral blood flow will not change
appreciably.
Mean arterial blood pressure will
slightly increase.
In a patient with heart failure the cardiac output is
5.0 L/min, the mean pulmonary artery pressure is
60 mm Hg, and the mean pulmonary artery wedge
pressure (left atrial pressure) is 30 mm Hg. If the
units used are L/min for CO, pulmonary vascular
resistance in PRU (Pulmonary Resistance Units) is:
A.
B.
C.
D.
A.
B.
Elevated at 12 PRU
Elevated at 6 PRU
Low at 0.17 PRU
Low at 4 PRU
A patient with coronary artery disease undergoes
coronary angiography, which reveals a 50%
decrease in the lumen of the anterior descending
coronary artery. For any arteriovenous coronary
pressure gradient, the flow through this artery
(compared to normal) will:
A.
B.
C.
D.
E.
6.
Cannot measure diastolic pressure.
Depends upon disappearance of
sound to measure systolic pressure.
May be falsely high in obese
patients.
May be falsely low with too narrow a
cuff.
Will not reflect changes produced by
exercise.
Be unaffected because of a reduction
in work load.
Be unaffected because of coronary
autoregulation.
Decrease by a factor of 16 times.
Decrease by a factor of 2.
Decrease by a factor of 8 times.
During administration of an unknown agent, systolic
pressure rises, ventricular end- diastolic volume is
unchanged, and the ventricular end-systolic volume
decreases. The drug was a:
A.
B.
Blood volume expansion.
Negative chronotropic agent.
C.
D.
E.
7.
When myocardial oxygen consumption (MVO2) is
increased two fold in the normal heart, the
myocardial blood flow will:
A.
B.
C.
D.
E.
8.
Placebo.
Positive inotropic agent.
Negative inotropic agent.
Approximately double.
Decrease by 50%.
Increase by 50%.
Increase in an exponential manner.
Increase only if perfusion pressure
rises.
Given the following data on a patient
Mean capillary hydrostatic pressure
30 mm Hg
Plasma osmotic pressure
28 mm Hg
Tissue hydrostatic pressure
5 mm Hg
Tissue osmotic pressure
8 mm Hg
There will be a net force (in mmHg) tending to move
water from the capillary to tissue of:
A.
B.
C.
D.
E.
11. Which of the following statements about the
Bernoulli effect is true?
A. It reflects changes only in the
components of total potential energy
B. With a uniform radius, velocity is
not uniform along the length of the
vessel.
C. If a vessel has a center diameter one
third of the diameter at either end, the
velocity within the narrow segment will
increase threefold.
D. Velocity is uniform while pressure
decreases are linear along the length of a
vessel not uniform in diameter.
E. Within a short vessel segment the
sum of kinetic and potential energy of a
blood vessel decrease markedly.
5
15
35
45
71
9.
If a patient has a cardiac arrest and the heart
has stopped for a sufficient amount of time, the arterial
and venous pressures will equilibrate. This is called the
mean:
A. Arterial pressure.
B. Circulatory filling pressure.
C. Hydrostatic refilling pressure.
D. Pulse pressure.
E. Transmural pressure.
10. For normal blood flow through a capillary bed,
from arteriole to venule, the ratio of post- to precapillary resistances must be :
A. less than .01.
B. maintained at 1.0.
C. less than 1.0 but greater than 0.1.
D. more than 1.0 but less than 10.
E. more than 10.
12. Which of these three curves
demonstrates the greatest
left ventricular compliance?
A
B
C
A patient has a systolic BP of 180 mmHg and a diastolic of
90 mmHg. If this patient has a heart rate of 100 bpm
what is the patient’s “double product”?
A. 1.2
B. 2.7
C. 8100
D. 9000
E. 12000
13. A patient with a mean systolic pressure of 101
mmHg and right atrial pressure of 1 mmHg has a
systemic vascular resistance of 25 mmHg/L. What is
this patient’s cardiac output in L/min?
A. 2.5
B. 4
C. 5
D. 25
Which of the following is the normal primary
determinate of peripheral (systemic) vascular
resistance?
E. Aortic vascular smooth muscle.
F. Central venous pressure.
G. Pre capillary arteriole sphincters.
H. Pulmonary capillary wedge
pressure.
I.
Right atrial pressure
14. A patient with a long history of alcohol abuse and a
poorly functioning liver presents to the ER with
peripheral edema. Of the following which is most
likely to cause this patient’s edema?
A.
B.
C.
D.
E.
Abdominal lymphatic pump
manipulation.
Deceased thoracic pressure.
Decreased capillary permeability.
Low levels of plasma albumin.
Thoracic lymphatic pump
manipulation.
15. A 22 yo male is found to have a blood pressure of
190/140 mmHg. Lab results show elevation of
plasma renin. Renal venous catheterization shows
an increased renin level in the right renal vein and a
decreased level of renin in the left renal vein. What
is the most likely cause of this patient’s
hypertension?
A.
A medical physiology exam.
B.
C.
D.
E.
An aldosterone secreting tumor.
Left renal arterial stenosis.
Pheochromocytoma.
Right renal arterial stenosis.
16. Coronary blood flow in the left coronary artery may
cease during
A. Atrial diastole.
B. Atrial systole.
C. Ventricular diastole.
D. Ventricular systole.
17. Which of the following statements best describes
cerebral vascular regulation?
A. Arterial blood pressure is
autoregulated from ~10 to 40
mmHg.
B. Brainstem vasculature exhibits the
most precise autoregulation.
C. Cerebral cortical vasculature has the
most precise autoregulatory ability.
D. The sagittal sinus contains
baroreceptors responsive to mean
cerebral vascular pressure.
E. Venous blood pressure is
autoregulated and remains at
greater than 10 mmHg.